Difference between revisions of "1957 AHSME Problems/Problem 40"

(created solution page)
 
(Solution)
Line 9: Line 9:
 
\textbf{(E)}\ \text{a negative irrational number}</math>
 
\textbf{(E)}\ \text{a negative irrational number}</math>
  
== Solution ==
+
== Solution 1 ==
<math>\boxed{\textbf{(D)}\text{ a positive or a negative irrational number}}</math>.
+
Note that if <math>y=-x^2+bx-8</math> has its vertex on the <math>x</math>-axis, then <math>-y=x^2-bx+8</math> will have its vertex on the x-axis as well. To find the location of the vertex of the parabola, we desire to put it in vertex form, where <math>-y=(x-h)^2+k</math>, and <math>(h,k)</math> is the location of the vertex. However, we know that <math>k=0</math>, because the vertex is on the <math>x</math>-axis. Thus, we know that <math>x^2-bx+8</math> must be the [[Perfect square#Perfect Square Trinomials|square of a linear term]]. Thus, <math>b=\pm 2 \cdot 1 \cdot \sqrt8 =\pm 4\sqrt2</math>, which are both irrational. Thus, our answer is <math>\boxed{\textbf{(D)}\text{ a positive or a negative irrational number}}</math>.
  
 
== See Also ==
 
== See Also ==

Revision as of 09:35, 27 July 2024

Problem

If the parabola $y = -x^2 + bx - 8$ has its vertex on the $x$-axis, then $b$ must be:

$\textbf{(A)}\ \text{a positive integer}\qquad \\  \textbf{(B)}\ \text{a positive or a negative rational number}\qquad \\  \textbf{(C)}\ \text{a positive rational number}\qquad\\  \textbf{(D)}\ \text{a positive or a negative irrational number}\qquad\\  \textbf{(E)}\ \text{a negative irrational number}$

Solution 1

Note that if $y=-x^2+bx-8$ has its vertex on the $x$-axis, then $-y=x^2-bx+8$ will have its vertex on the x-axis as well. To find the location of the vertex of the parabola, we desire to put it in vertex form, where $-y=(x-h)^2+k$, and $(h,k)$ is the location of the vertex. However, we know that $k=0$, because the vertex is on the $x$-axis. Thus, we know that $x^2-bx+8$ must be the square of a linear term. Thus, $b=\pm 2 \cdot 1 \cdot \sqrt8 =\pm 4\sqrt2$, which are both irrational. Thus, our answer is $\boxed{\textbf{(D)}\text{ a positive or a negative irrational number}}$.

See Also

1957 AHSC (ProblemsAnswer KeyResources)
Preceded by
Problem 39
Followed by
Problem 41
1 2 3 4 5 6 7 8 9 10 11 12 13 14 15 16 17 18 19 20 21 22 23 24 25 26 27 28 29 30 31 32 33 34 35 36 37 38 39 40 41 42 43 44 45 46 47 48 49 50
All AHSME Problems and Solutions

The problems on this page are copyrighted by the Mathematical Association of America's American Mathematics Competitions. AMC logo.png